Trigonomic Integration question

  • Thread starter Thread starter jordan123
  • Start date Start date
  • Tags Tags
    Integration
Click For Summary
The integral of (1+x)/(1+x^2) dx can be approached by splitting it into two separate integrals: ∫(1/(1+x^2)) dx and ∫(x/(1+x^2)) dx. The first integral is straightforward and corresponds to an arctangent function, while the second can be solved using a simple substitution. Participants in the discussion confirm this method and emphasize the importance of rewriting the integrand into manageable parts. Overall, the discussion focuses on clarifying the steps needed to solve the integral effectively. Understanding these techniques is essential for tackling similar trigonometric integration problems.
jordan123
Messages
15
Reaction score
0

Homework Statement


Sorry for the lack of latex.

The question is this.

Integral of ((1+x)/(1+(x^2))) dx

If someone could put that together on latex, that would be awesome!


The Attempt at a Solution


Im not exactly sure what to do, at all. So really any help is great. Even just a hint and I will probably get it. Thanks!
 
Physics news on Phys.org
Split into two integrals.
\int \frac{1 + x}{1 + x^2}dx
= \int \frac{1}{1 + x^2} dx + \int \frac{x}{1 + x^2} dx

The first is pretty straightforward. The second requires only an ordinary substitution.
 
((1+x)/(1+(x^2))) dx = (1/(1+(x^2)) dx+(x)/(1+(x^2)) dx

Yes, first part is trigonometric and second is log..
 
Do you mean

<br /> <br /> \int \frac{1 + x} {1 + x^2} dx<br /> <br />

Re-write the integrand as two terms
 
Mark44 said:
Split into two integrals.
\int \frac{1 + x}{1 + x^2}dx
= \int \frac{1}{1 + x^2} dx + \int \frac{x}{1 + x^2} dx

The first is pretty straightforward. The second requires only an ordinary substitution.

lol, thanks.
 
Question: A clock's minute hand has length 4 and its hour hand has length 3. What is the distance between the tips at the moment when it is increasing most rapidly?(Putnam Exam Question) Answer: Making assumption that both the hands moves at constant angular velocities, the answer is ## \sqrt{7} .## But don't you think this assumption is somewhat doubtful and wrong?

Similar threads

  • · Replies 9 ·
Replies
9
Views
2K
  • · Replies 1 ·
Replies
1
Views
2K
  • · Replies 105 ·
4
Replies
105
Views
6K
  • · Replies 12 ·
Replies
12
Views
2K
  • · Replies 11 ·
Replies
11
Views
2K
  • · Replies 2 ·
Replies
2
Views
1K
  • · Replies 22 ·
Replies
22
Views
3K
  • · Replies 9 ·
Replies
9
Views
2K
Replies
24
Views
3K
Replies
7
Views
2K